Difference between revisions of "2018 AMC 12B Problems/Problem 6"

(Also See)
(Problem)
Line 1: Line 1:
 
==Problem==
 
==Problem==
 +
 +
Suppose <math>S</math> cans of soda can be purchased from a vending machine for <math>Q</math> quarters. Which of the following expressions describes the number of cans of soda that can be purchased for <math>D</math> dollars, where 1 dollar is worth 4 quarters?
  
 
==Solution==
 
==Solution==

Revision as of 16:58, 16 February 2018

Problem

Suppose $S$ cans of soda can be purchased from a vending machine for $Q$ quarters. Which of the following expressions describes the number of cans of soda that can be purchased for $D$ dollars, where 1 dollar is worth 4 quarters?

Solution

See Also

2018 AMC 12B (ProblemsAnswer KeyResources)
Preceded by
Problem 5
Followed by
Problem 7
1 2 3 4 5 6 7 8 9 10 11 12 13 14 15 16 17 18 19 20 21 22 23 24 25
All AMC 12 Problems and Solutions

The problems on this page are copyrighted by the Mathematical Association of America's American Mathematics Competitions. AMC logo.png